In: Finance
Consider the three stocks in the following table.
Ptrepresents price at time t, and
Qtrepresents shares outstanding at time
t. There are three time points 0, 1, and 2.
| P0 | Q0 | P1 | Q1 | P2 | Q2 | |
| A | 90 | 100 | 95 | 100 | 100 | 100 |
| B | 50 | 200 | 45 | 200 | 45 | 200 |
| C | 100 | 200 | 110 | 200 | 105 | 200 |
a) Form an equally weighted portfolio in stocks A, B, C at time 0
and hold it to time 1.
Calculate the portfolio return from 0 to 1.
Keep two decimal places.
b) Form an equally weighted portfolio in stocks B, C (No Stock A) at time 1 and hold it to time 2.
Calculate the portfolio return from 1 to 2.
Keep two decimal places.
Solution (a)
| Stock | Weights | P0 | Q0 | Purchase Cost | P1 | Q1 | Sale Value | Return | Return * Weight |
| A | 0.33 | 90.00 | 100.00 | 9,000.00 | 95.00 | 100.00 | 9,500.00 | 500.00 | 166.67 |
| B | 0.33 | 50.00 | 200.00 | 10,000.00 | 45.00 | 200.00 | 9,000.00 | -1,000.00 | -333.33 |
| C | 0.33 | 100.00 | 200.00 | 20,000.00 | 110.00 | 200.00 | 22,000.00 | 2,000.00 | 666.67 |
| Portfolio Return | 500.00 |
Return in %
Stock A = 166.67 / 9,000 * 100 = 1.85%
Stock B = -333.33 / 10,000 * 100 = -3.33%
Stock C = 666.67 / 20,000 * 100 = -3.33%
Total Portfolio Return = 1.85% (1.85% + (-3.33%) + 3.33%)
Solution (b)
| Stock | Weights | P1 | Q1 | Purchase Cost | P2 | Q2 | Sale Value | Return | Return * Weight |
| B | 0.50 | 45.00 | 200.00 | 9,000.00 | 45.00 | 200.00 | 9,000.00 | - | - |
| C | 0.50 | 110.00 | 200.00 | 22,000.00 | 105.00 | 200.00 | 21,000.00 | -1,000.00 | -500.00 |
| Portfolio Return | -500.00 |
Return in %
Stock B = 0 / 9,000 * 100 = 0%
Stock C = -500 / 22,000 * 100 = -2.27%
Total Portfolio Return = -2.27% (0% + (-2.27%))